0 Daumen
618 Aufrufe

Aufgabe: 

Zeige mit Induktion dass:$$\sum_{i=1}^{n} i^{3}=\left(\sum_{i=1}^{n} i\right)^{2}=\frac{n^{2}(n+1)^{2}}{4}.$$


Problem/Ansatz:

Muss ich den ersten Term  und den zweiten Term zeigen und erst dann den ersten mit dem dritten ? 
Oder wie muss ich hier vorgehen  ?

Avatar von

Wurde vorher schon mal bewiesen, dass 1+2+...+(n-1)+n=n(n+1)/2 ist? Das würde im hinteren Teil ja einfach nur diese Gleichung quadriert und der eigentlich Induktionsbeweis müsste nur vom 1. zum 2. oder dritten Term gehen.

4 Antworten

+1 Daumen

Ich würde den ersten mit dem dritten und den zweiten mit dem dritten Term zeigen. Wenn das gilt gilt auch das der erste Term gleich dem zweiten Term ist.

Brauchst du selber bei der Induktion Hilfe oder schaffst du das so?

Avatar von 477 k 🚀

Hilfe: Ist das richtig ? 

Induktionsanfang: 
Zeige ich es für n = 1. 

Induktionsverankerung:
Hier weiss ich nicht was effektiv tun? Ausser zu schreiben, Für ein beliebiges aber festes n gilt die Aussage. 

Induktionsschluss:
Ich zeige, dass ich von n nach n+1 komme.

Die Induktionsverankerung ist in diesem Fall das es für n = 1 gilt. Da brauchst du nur 1 einseten und zeigen das überall das gleiche (1) heraus kommt.

Induktionsschluss oder schritt ist richtig. Wenn es für n gilt zeigst du, dass es auch für n + 1 gilt.

+1 Daumen

nehmen wir mal als bewiesen, dass \(\sum_{i=1}^{n}{i}=\frac{n(n-1)}{2}\) und \(\sum_{i=1}^{n} i^{3}=\frac{n^{2}(n-1)^{2}}{4}\), dann weißt du, dass \(\left(\sum_{i=1}^{n}{i}\right)^2=\sum_{i=1}^{n} i^{3}\)

Also beweise \(\sum_{i=1}^{n} i^{3}=\frac{n^2(n-1)^2}{4}\).

Vice versa ist natürlich auch legitim.

Avatar von 28 k

"Also beweise..." Du hast dich da vertippt, sollte ein + kein - sein :)

Richtig. Noch öfter passiert es mir, bei der geomtrischen Reihe den Anfangswert nicht zu beachten, nervig!

+1 Daumen

Aloha :)

Verankerung bei \(n=1\):$$\sum\limits_{i=1}^1i^3=1\quad;\quad\frac{1^2\cdot(1+1)^2}{4}=\frac{4}{4}=1\quad\checkmark$$

Induktionssschritt \(n\to n+1\):

$$\sum\limits_{i=1}^{n+1}i^3=\sum\limits_{i=1}^ni^3+(n+1)^3\;\stackrel{I.V.}{=}\;\frac{n^2(n+1)^2}{4}+(n+1)^3=\frac{n^2(n+1)^2}{4}+\frac{4(n+1)^3}{4}$$$$=\frac{n^2(n+1)^2}{4}+\frac{4(n+1)(n+1)^2}{4}=\frac{(n^2+4n+4)(n+1)^2}{4}=\frac{(n+2)^2(n+1)^2}{4}$$$$=\frac{(n+1)^2(n+2)^2}{4}\quad\checkmark$$Nach dem Induktionsschritt erhältst du die zu zeigende Summenformel, allerdings mit \(n+1\) statt mit \(n\).

Avatar von 148 k 🚀
0 Daumen
Avatar von 162 k 🚀

Ein anderes Problem?

Stell deine Frage

Willkommen bei der Mathelounge! Stell deine Frage einfach und kostenlos

x
Made by a lovely community